Stats
Stats
Stats
1. (20 points) For each of the following statements, circle whether it is true or false. You need
not justify your answers.
(a) True or False : The sum of two independent gamma distributions has a gamma distri-
bution.
(b) True or False : If X1 ∼ Gamma(α1 , λ) and X2 ∼ Gamma(α2 , λ), then X1 + X2 ∼
Gamma(α1 + α2 , λ).
X X
(c) True or False: If X and Y are independent N (0, 1) random variables then Y and |Y |
have the same distribution.
(d) True or False: The finite population correction factor gets applied when sampling a
finite population without replacement.
iid
(e) True or False: If X1 , . . . , Xn ∼ N (µ, σ 2 ), then S 2 is an unbiased estimator of σ 2 , but
Var(σ̂ 2 ) < Var(S 2 ).
(f) True or False: Suppose θb is an estimator of θ with E(θb2 ) < ∞ and T is a sufficient
b ) ̸= θ,
statistic for θ. If θ̃ = E(θ|T b then MSE(θ̃) < MSE(θ). b
(g) True or False : The variance of an estimator of θ cannot be less than the CRLBθ .
(h) True or False: The variance of an unbiased estimator of θ cannot be less than the
CRLBθ .
(i) True or False : The probability that the null hypothesis is falsely rejected is equal to
the power of the test.
(j) True or False: In comparing two simple hypotheses, H0 and Ha , with significance level
α, no other test will have more power than the likelihood ratio test.
Solution:
(a) The distributions should also have the same rate parameter λ.
(c) One approach: Show X/Y and X/|Y | have the same the cdf by conditioning on the
sign of Y . The fourth equals sign is justified because X and −X have the same
distribution.
P (X/Y ≤ z) = P (X/Y ≤ z|Y > 0)P (Y > 0) + P (X/Y ≤ z|Y < 0)P (Y < 0)
= P (X ≤ zY |Y > 0) · 1/2 + P (X ≥ zY |Y < 0) · 1/2
= P (X ≤ zY |Y > 0) · 1/2 + P (−X ≤ −zY |Y < 0) · 1/2
= P (X ≤ zY |Y > 0) · 1/2 + P (X ≤ −zY |Y < 0) · 1/2
= P (X ≤ z|Y |) · 1/2 + P (X ≤ z|Y |) · 1/2
= P (X ≤ z|Y |)
= P (X/|Y | ≤ z).
This is the Cauchy distribution (which is also a t distribution with 1 degree of free-
dom).
S&DS 242 Midterm Exam Solutions
(d) Of course!
(e)
2
2 n−1 2 n−1
Var(σ̂ ) = Var S = Var(S 2 ) < Var(S 2 )
n n
b ) ̸= θ,
(f) If θ̃ = E(θ|T b the inequality in the Rao-Blackwell Theorem is strict.
Page 2 of 8
S&DS 242 Midterm Exam Solutions
Solution:
Z 1
µ1 = E(X) = x(1 + (2x − 1)θdx
0
Z 1
= (2θx2 + (1 − θ)x)dx
0
2θ 1 − θ θ+3
= + = .
3 2 6
θb = 6X − 3.
(b) Find the mean squared error of the estimator in part (a).
Solution: We first compute the second moment of the distribution to get the variance.
Z 1
2
E(X ) = x2 (1 + (2x − 1)θdx
0
Z 1
= (2θx3 + (1 − θ)x2 )dx
0
θ 1−θ θ+2
= + = .
2 3 6
And so
θ+3 2
2 θ+2 2
Var(X) = E(X ) − (EX) = −
6 6
2
θ + 2 θ + 6θ + 9 3−θ 2
= − = .
6 36 36
The MSE is
Page 3 of 8
S&DS 242 Midterm Exam Solutions
Solution: Z 1
θ+1
µ1 = E(X) = x(θ + 1)xθ dx = .
0 θ+2
Solving for θ,
µ1 (θ + 2) = θ + 1
µ1 θ − θ = 1 − 2µ1
1 − 2µ1
θ= .
µ1 − 1
1 − 2X
θb = .
X −1
(b) Show that this distribution is in the exponential family. Use this to give a sufficient
statistic for θ.
Solution: Since
we see that this distribution is in the exponential family with c(θ) = θ, T (x) = log x,
d(θ) = log(θ + 1) and S(x) = 0, with A = [0, 1].
Therefore
Xn n
X
T (Xi ) = log Xi
i=1 i=1
is sufficient for θ.
Page 4 of 8
S&DS 242 Midterm Exam Solutions
Solution:
n
!
∂ ∂ X
log f (x1 , . . . , xn |θ) = n log(θ + 1) + θ log xi
∂θ ∂θ
i=1
n
(θ + 1) ni=1 log xi + n
P
n X
= + log xi = = 0.
θ+1 θ+1
i=1
1 (θ+1)2
And so the asymptotic variance is nI(θ) = n .
Page 5 of 8
S&DS 242 Midterm Exam Solutions
1 −3 2 −x/β
f (x) = 2β x e , if x > 0,
0, if x ≤ 0.
1
Note that this a gamma distribution with α = 3 and λ = β. You may use the following facts
about this distribution:
(i) E(X) = 3β (ii) Var(X) = 3β 2 (iii) the mgf is MX (t) = (1 − βt)−3
(a) Find the mle of β.
Solution:
n n
!
∂ ∂ X 1X
log f (x1 , . . . , xn |θ) = −n log 2 − 3n log β + 2n log xi − xi
∂θ ∂θ β
i=1 i=1
n Pn
3n 1 i=1 xi − 3nβ
X
=− + 2 xi = = 0.
β β β2
i=1
Solution: Since
b = 1 E(X) = 1 · 3β = β,
E(β)
3 3
βb is unbiased. Also,
2 2
b = 1 Var(X) = 3β = β → 0 as n → ∞.
Var(β)
9 9n 3n
Page 6 of 8
S&DS 242 Midterm Exam Solutions
1 −3 2 −x/β
f (x) = 2β x e , if x > 0,
0, if x ≤ 0.
Note that this a gamma distribution with α = 3 and λ = β1 . You may use the following
facts about this distribution:
(i) E(X) = 3β (ii) Var(X) = 3β 2 (iii) the mgf is MX (t) = (1 − βt)−3
(c) Show that W = β2 ni=1 Xi has a chi-squared distribution with 6n degrees of freedom.
P
Hint: Use moment generating functions.
Pn
Solution: For notation simplicity, Sn denote i=1 Xi . We have Sn ∼ Gamma(α =
3n, λ = 1/β). The mgf of W is
Wt
2
S t
β n
Sn β2 t
2t
MW (t) = E(e )=E e =E e = MSn
β
3n 6n/2
1/β 1
= = = (1 − 2t)−6n/2 ,
1β − 2t/β 1 − 2t
(d) Use your answer from (c) to derive a two-sided 90% confidence interval for β.
Note: Your answer should in terms of χ2n (α) numbers, where χ2n (α) is the value where the
right tail has probability α. For example, if W ∼ χ2n (where n is the degrees of freedom),
then P (W > χ2n (α)) = α.
2 ni=1 Xi 2 ni=1 Xi
P P
≤ β ≤ .
χ26n (0.05) χ26n (0.95)
2nX 2nX
≤β≤ .
χ26n (0.05) χ26n (0.95)
Page 7 of 8
S&DS 242 Midterm Exam Solutions
5. (16 points) Under H0 , a random variable has pdf f (x) = 2x for 0 ≤ x ≤ 1. Under Ha , it has
pdf f (x) = 3x2 for 0 ≤ x ≤ 1.
(a) Find the best test (in terms of power) of H0 versus Ha that has significance level α = 0.19.
What is the power of this test?
Solution: By the Neyman-Pearson Lemma, the powerful test is the likelihood ratio
2x 2
test. The likelihood ratio is 3x 2 = 3x , which rejects H0 for large x. Let x0 denote the
boundary of the rejection region, so that we reject H0 if x ∈ [x0 , 1] and fail to reject
if x ∈ [0, x0 ).
To achieve a significance level of α = 0.19, we must have
Z 1 1
2x dx = x = 1 − x20 .
2
0.19 =
x0 x0
(b) Assume the two hypotheses have equal prior probability. What is the rejection region?
What is the significance level? What is the power of the test?
2
Solution: In this case reject when the likelihood ratio 3x ≤ 1. Solving for x gives
2 we
the rejection region 3 , 1 .
The significance level is
Z 1 1
2
4 5
α= 2x dx = x = 1 − = .
2/3 2/3 9 9
Page 8 of 8